Indian Mathematical Olympiad 2018

Geometry Level 4

Let A B C ABC be a non-equilateral triangle with integer sides. Let D D and E E be respectively the mid-points of B C BC and C A CA . Let G G be the centroid of triangle A B C ABC . Suppose D , C , E , G D, C, E, G are concyclic. Find the least possible perimeter of triangle A B C ABC .


The answer is 37.

This section requires Javascript.
You are seeing this because something didn't load right. We suggest you, (a) try refreshing the page, (b) enabling javascript if it is disabled on your browser and, finally, (c) loading the non-javascript version of this page . We're sorry about the hassle.

1 solution

Mark Hennings
Feb 1, 2018

Suppose that the points A , B , C A,B,C are represented by complex numbers a , b , 0 a,b,0 respectively. Without loss of generality, we can assume that a a is a positive real. We therefore also require that u = a = a u = |a| = a , v = b v =|b| and w = a b w = |a-b| are positive integers. We can also assume that u v u \ge v .

The fact that C , D , E , G C,D,E,G are concyclic means that the four complex numbers 0 , 1 2 a , 1 2 b , 1 3 ( a + b ) 0,\tfrac12a,\tfrac12b,\tfrac13(a+b) are concyclic. This means that their cross-ratio is real, and hence b ( 2 b a ) 2 ( b 2 a 2 ) = 1 2 b ( 1 3 b 1 6 a ) 1 2 ( b a ) 1 3 ( b + a ) = 1 2 λ \frac{b(2b-a)}{2(b^2 - a^2)} \; = \; \frac{\frac12b (\frac13b - \frac16a)}{\frac12(b-a)\frac13(b+a)} \; = \; \tfrac12\lambda for some real λ \lambda . This implies that ( 2 λ ) b 2 a b + λ a 2 = 0 (2-\lambda)b^2 - ab + \lambda a^2 \; = \; 0 Since we want A B C ABC to be a nontrivial triangle, b b cannot be a real multiple of a a , and so we deduce that 4 λ ( 2 λ ) > 1 4\lambda(2-\lambda) > 1 , with b = a 2 ( 2 λ ) { 1 ± i 4 λ ( 2 λ ) 1 } b \; = \; \frac{a}{2(2-\lambda)}\left\{ 1 \pm i\sqrt{4\lambda(2-\lambda) - 1}\right\} From this we deduce that u = a v = a λ 2 λ w = a 2 λ u \; = \; a \hspace{2cm} v \; = \; a\sqrt{\tfrac{\lambda}{2-\lambda}} \hspace{2cm} w \; = \; \tfrac{a}{\sqrt{2-\lambda}} From this we deduce that λ = v 2 w 2 \lambda = \tfrac{v^2}{w^2} and hence that u 2 + v 2 = 2 w 2 u^2 + v^2 \; = \; 2w^2 Since u 2 + v 2 u^2 + v^2 is even, u u and v v must have the same parity. If we define nonnegative integers p , q p,q by setting u + v = 2 p u+v = 2p , u v = 2 q u-v=2q , we deduce that p 2 + q 2 = w 2 p^2 + q^2 = w^2 . We therefore need to consider Pythagorean triples.

  • The first case is p = 1 p=1 , q = 0 q=0 , w = 1 w=1 , which leads to u = v = w = 1 u=v=w=1 . This triangle is equilateral, and so should be ignored.
  • The second case is p = 4 p=4 , q = 3 q=3 , w = 5 w=5 , which leads to u = 7 u=7 , v = 1 v=1 . A triangle with sides 7 , 1 , 5 7,1,5 does not exist, so we ignore this case as well.
  • The third case is p = 12 p=12 , q = 5 q=5 , w = 13 w=13 , which leads to u = 17 u=17 , v = 7 v=7 . A triangle with sides 17 , 7 , 13 17,7,13 does exist, so it must be the desired triangle with shortest perimeter. This makes the answer 17 + 7 + 13 = 37 17 + 7 + 13 = \boxed{37} .

Did exactly the same

A Former Brilliant Member - 3 years, 4 months ago

0 pending reports

×

Problem Loading...

Note Loading...

Set Loading...